Difference between revisions of "1964 AHSME Problems/Problem 3"

(Solution $1$=)
Line 13: Line 13:
 
==Solution 1==
 
==Solution 1==
  
By the definition of quotient and remainder, problem states that <math>x = uy + v</math>.
+
*We can solve this problem by elemetary modular arthmetic,
 +
<math>x</math> \equiv. <math>v</math> (<math>mod y</math>) <math>=></math> <math>x</math> + <math>2uy</math> \equiv. <math>v</math> (<math>mod y</math>).
  
The problem asks to find the remainder of <math>x + 2uy</math> when divided by <math>y</math>.  Since <math>2uy</math> is divisible by <math>y</math>, adding it to <math>x</math> will not change the remainder.  Therefore, the answer is <math>\boxed{\textbf{(D)}}</math>.
+
% Solution by GEOMETRY-WIZARD<math>
 +
==Solution 2==
 +
 
 +
By the definition of quotient and remainder, problem states that </math>x = uy + v<math>.
 +
 
 +
The problem asks to find the remainder of </math>x + 2uy<math> when divided by </math>y<math>.  Since </math>2uy<math> is divisible by </math>y<math>, adding it to </math>x<math> will not change the remainder.  Therefore, the answer is </math>\boxed{\textbf{(D)}}<math>.
  
==Solution 2==
+
==Solution 3==
If the statement is true for all values of <math>(x, y, u, v)</math>, then it must be true for a specific set of <math>(x, y, u, v)</math>.
+
If the statement is true for all values of </math>(x, y, u, v)<math>, then it must be true for a specific set of </math>(x, y, u, v)<math>.
  
If you let <math>x=43</math> and <math>y = 8</math>, then the quotient is <math>u = 5</math> and the remainder is <math>v = 3</math>.  The problem asks what the remainder is when you divide <math>x + 2uy = 43 + 2 \cdot 5 \cdot 8 = 123</math> by <math>8</math>.  In this case, the remainder is <math>3</math>.
+
If you let </math>x=43<math> and </math>y = 8<math>, then the quotient is </math>u = 5<math> and the remainder is </math>v = 3<math>.  The problem asks what the remainder is when you divide </math>x + 2uy = 43 + 2 \cdot 5 \cdot 8 = 123<math> by </math>8<math>.  In this case, the remainder is </math>3<math>.
  
When you plug in <math>u=5</math> and <math>v = 3</math> into the answer choices, they become <math>0, 5, 10, 3, 6</math>, respectively.  Therefore, the answer is <math>\boxed{\textbf{(D)}}</math>.
+
When you plug in </math>u=5<math> and </math>v = 3<math> into the answer choices, they become </math>0, 5, 10, 3, 6<math>, respectively.  Therefore, the answer is </math>\boxed{\textbf{(D)}}$.
  
 
==See Also==
 
==See Also==

Revision as of 05:39, 31 December 2023

Problem

When a positive integer $x$ is divided by a positive integer $y$, the quotient is $u$ and the remainder is $v$, where $u$ and $v$ are integers. What is the remainder when $x+2uy$ is divided by $y$?

$\textbf{(A)}\ 0 \qquad \textbf{(B)}\ 2u \qquad \textbf{(C)}\ 3u \qquad \textbf{(D)}\ v \qquad \textbf{(E)}\ 2v$


Solution 1

  • We can solve this problem by elemetary modular arthmetic,

$x$ \equiv. $v$ ($mod y$) $=>$ $x$ + $2uy$ \equiv. $v$ ($mod y$).

% Solution by GEOMETRY-WIZARD$==Solution 2==

By the definition of quotient and remainder, problem states that$ (Error compiling LaTeX. Unknown error_msg)x = uy + v$.

The problem asks to find the remainder of$ (Error compiling LaTeX. Unknown error_msg)x + 2uy$when divided by$y$.  Since$2uy$is divisible by$y$, adding it to$x$will not change the remainder.  Therefore, the answer is$\boxed{\textbf{(D)}}$.

==Solution 3== If the statement is true for all values of$ (Error compiling LaTeX. Unknown error_msg)(x, y, u, v)$, then it must be true for a specific set of$(x, y, u, v)$.

If you let$ (Error compiling LaTeX. Unknown error_msg)x=43$and$y = 8$, then the quotient is$u = 5$and the remainder is$v = 3$.  The problem asks what the remainder is when you divide$x + 2uy = 43 + 2 \cdot 5 \cdot 8 = 123$by$8$.  In this case, the remainder is$3$.

When you plug in$ (Error compiling LaTeX. Unknown error_msg)u=5$and$v = 3$into the answer choices, they become$0, 5, 10, 3, 6$, respectively.  Therefore, the answer is$\boxed{\textbf{(D)}}$.

See Also

1964 AHSC (ProblemsAnswer KeyResources)
Preceded by
Problem 2
Followed by
Problem 4
1 2 3 4 5 6 7 8 9 10 11 12 13 14 15 16 17 18 19 20 21 22 23 24 25 26 27 28 29 30 31 32 33 34 35 36 37 38 39 40
All AHSME Problems and Solutions

The problems on this page are copyrighted by the Mathematical Association of America's American Mathematics Competitions. AMC logo.png